หัวข้อ: Warm Up for POSN Camp#2
ดูหนึ่งข้อความ
  #100  
Old 24 มีนาคม 2013, 00:36
tonklaZolo's Avatar
tonklaZolo tonklaZolo ไม่อยู่ในระบบ
กระบี่ไว
 
วันที่สมัครสมาชิก: 31 กรกฎาคม 2011
ข้อความ: 223
tonklaZolo is on a distinguished road
Default

อ้างอิง:
ข้อความเดิมเขียนโดยคุณ Thgx0312555 View Post
Combi สักข้อ
ให้ $a_1,a_2,...,a_m$ เป็นสมาชิกที่แตกต่างกันของ $\left\{ 1,2,...,n \right\}$ ซึ่งมีคุณสมบัติคือ
สำหรับ $1\le i,j \le m$ ถ้า $a_i+a_j \le n$ จะมี $1 \le k \le m$ ซึ่ง $a_k=a_i+a_j$

จงพิสูจน์ว่า
$\dfrac{a_1+a_2+\cdots +a_m}{m} \ge \dfrac{n+1}{2}$
ข้อ1 วันแรก IMO 1994
__________________
WHAT MAN BELIEVES
MAN CAN ACHIEVE
ตอบพร้อมอ้างอิงข้อความนี้